Math, asked by suriyamammu9802, 5 months ago

A random variable X has the following probability distribution X 0 1 2 3 4 5 6 P(X) K 3k 5k 7k 9k 11k 13k (i) Find k (ii) P(X < 4) , P(X ≥ 5), P(3 < X ≤ 6) (iii) What will be the minimum value of K so that P(X ≤ 2) >0.3

Answers

Answered by MaheswariS
2

A random variable X has the following probability distribution X 0 1 2 3 4 5 6 P(X) K 3k 5k 7k 9k 11k 13k (i) Find k (ii) P(X < 4) , P(X ≥ 5), P(3 < X ≤ 6) (iii) What will be the minimum value of K so that P(X ≤ 2) >0.3

\textbf{Given:}

\mathsf{\;\;X\;\;\;0\;\;1\;\;2\;\;3\;\;4\;\;5\;\;5\;\;6}

\mathsf{P(X)\;\;k\;3k\;5k\;7k\;9k\;11k\;13k\;}

\textbf{To find:}

\mathsf{(i)The\;value\;of\;k}

\mathsf{(ii)P(X&lt;4),\;P(x\geq\,5),\;P(3\,&lt;X&lt;\leq\,6)}

\textsf{(iii)The minimum value of X so that}\;\mathsf{P(X\,\leq\,2)&gt;0.3}

\textbf{Solution:}

\textsf{(i) We know that, sum of the probabilities is 1}

\implies\mathsf{k+3k+5k+7k+9k+11k+19k=1}

\implies\mathsf{55k=1}

\implies\mathsf{k=\dfrac{1}{55}}

\mathsf{(ii)}

\mathsf{P(X&lt;4)=P(x=0)+P(x=1)+P(x=2)+P(x=3)}

\mathsf{P(X&lt;4)=k+3k+5k+7k=16k=\dfrac{16}{55}}

\implies\boxed{\mathsf{P(X&lt;4)=\dfrac{16}{55}}}

\mathsf{P(X\,\geq\,5)=P(x=5)+P(x=6)}

\mathsf{P(X\,\geq\,5)=11k+13k=24k=\dfrac{24}{55}}

\implies\boxed{\mathsf{P(X\,\geq\,5)=\dfrac{24}{55}}}

\mathsf{P(3&lt;X\leq\,6)=P(x=4)+P(x=5)+P(x=6)}

\mathsf{P(3&lt;X\leq\,6)=9k+11k+13k=33k=\dfrac{33}{55}=\dfrac{3}{5}}

\implies\boxed{\mathsf{P(3&lt;X\leq\,6)=\dfrac{3}{5}}}

\mathsf{(iii)}

\mathsf{For\,X=0,\;P(0)=\dfrac{1}{55}=0.02}

\mathsf{For\,X=1,\;P(1)=\dfrac{3}{55}=0.06}

\mathsf{For\,X=2,\;P(2)=\dfrac{5}{55}=0.1}

\textsf{There is no X satisfying the given condition}

Similar questions